Evaluating Volume Integrals and Divergence Theorm

In summary, the integral is either a volume integral of a surface integral, whichever is easier. I believe it would be easier to do as a volume integral though I am honestly not sure saying I am not really understand how things like line integrals and surface integrals work in the sense of things like Stokes Theorm and Div. Theorm.
  • #1
Thadis
44
0

Homework Statement


Evaluate the integral as either a volume integral of a surface integral, whichever is easier.

[itex]\iiint \nabla .F\,d\tau[/itex] over the region [itex]x^2+y^2+z^2 \leq 25[/itex], where [itex]F=(x^2+y^2+z^2)(x*i+y*j+z*k)[/itex]

Homework Equations


[itex]\iiint \nabla .F\,d\tau =\iint F.n\,d\sigma[/itex]


The Attempt at a Solution


I believe this would be easier to do as a volume integral though I am honestly not sure saying I am not really understand how things like line integrals and surface integrals work in the sense of things like Stokes Theorm and Div. Theorm.

For the volume intregral I did just integrated the Div(F) from -5 to 5 for all three integrals saying that is the range each of the variables will go through. I plugged this into Wolfram Alpha and got an answer of 25000 though I am not sure if I did this question correctly.

If I wanted to do a solve this using the right hand side of the Div. Theorem how would I approach this?
 
Physics news on Phys.org
  • #2
Thadis said:

Homework Statement


Evaluate the integral as either a volume integral of a surface integral, whichever is easier.

[itex]\iiint \nabla .F\,d\tau[/itex] over the region [itex]x^2+y^2+z^2 \leq 25[/itex], where [itex]F=(x^2+y^2+z^2)(x*i+y*j+z*k)[/itex]

Homework Equations


[itex]\iiint \nabla .F\,d\tau =\iint F.n\,d\sigma[/itex]


The Attempt at a Solution


I believe this would be easier to do as a volume integral though I am honestly not sure saying I am not really understand how things like line integrals and surface integrals work in the sense of things like Stokes Theorm and Div. Theorm.

For the volume intregral I did just integrated the Div(F) from -5 to 5 for all three integrals saying that is the range each of the variables will go through.

If you let all three variables go from -5 to 5 you are describing a region shaped like a rectangular block. You have a sphere.
 
  • #3
Thadis said:
For the volume intregral I did just integrated the Div(F) from -5 to 5 for all three integrals saying that is the range each of the variables will go through. I plugged this into Wolfram Alpha and got an answer of 25000 though I am not sure if I did this question correctly.
That's not what I got from doing what you said you did, which suggests you didn't calculate div F correctly. Show us how you calculated div F.

You may want to switch to spherical coordinates to do the integrals.
 
  • #4
LCKurtz said:
If you let all three variables go from -5 to 5 you are describing a region shaped like a rectangular block. You have a sphere.

Ahh, I knew I was forgetting something. For this case once I converted x,y, and z into sphrerical coordinates wouldn't the limits of integration be r=0 to r=5, θ= 0 to 2pi, and [itex]\phi[/itex]=0 to 2pi? Also just to double check. Would taking the volume integral actually easier in this case?
 
  • #5
I was just thinking about it but would actually turning it into a surface integral make it easier? Saying you would have the double integral of the force dotted into the normal, which would give out [itex](x^2+y^2+z^2)^2/5[/itex] since the normal to the sphere is just the normalized position vector? And since this is only going to be on the surface of the sphere we know that [itex](x^2+y^2+z^2)=r=5[/itex] . So would the answer then be 20pi^2? Since I would be able to draw out a consant 5, and then all that would be left would to do the integral that has no terms left inside with the limits of the two intergals both being from 0 to 2pi?
 
  • #6
The answer is ##12500\pi##.
 
  • #7
Thadis said:
I was just thinking about it but would actually turning it into a surface integral make it easier? Saying you would have the double integral of the force dotted into the normal, which would give out [itex](x^2+y^2+z^2)^2/5[/itex] since the normal to the sphere is just the normalized position vector?
You're saying the integral is equal to that, or F dotted into the normal? Where did the 5 come from?

And since this is only going to be on the surface of the sphere we know that [itex](x^2+y^2+z^2)=r=5[/itex].
##r = \sqrt{x^2+y^2+z^2}##

So would the answer then be 20pi^2? Since I would be able to draw out a constant 5, and then all that would be left would to do the integral that has no terms left inside with the limits of the two integrals both being from 0 to 2pi?
Those limits aren't correct. The polar angle doesn't go from 0 to ##2\pi##.
 
  • #8
vela said:
You're saying the integral is equal to that, or F dotted into the normal? Where did the 5 come from?
Sorry, was typing that in a hurry saying I was having to leave right then.

What I was trying to say, since the normal to the sphere is the normalized position vector, which would be [itex]{x,y,z}/5[/itex] since we know the distance to the surface is 5. If we dot this into the force that would create, [itex](x^2+y^2+z^2)*{x,y,z}.{x,y,z}/5[/itex] after doing the dot product turns into [itex](x^2+y^2+z^2)^2/5[/itex].

After this I did mess up and say (x^2+y^2+z^2)=5 where I should of said it is equal to 25. So that means the integrad is equal to 25^2/5?

And since the azimuthal angle is rotating completely around that means the polar angle only actually needs to go from 0 to pi correct?

I must be making some sort of mistake somewhere saying the answer I am getting is only like 250pi. Is there anything that you can see that I am doing wrong? Or is my entire train of thought forgetting a crucial point?
 
  • #9
What did you use for ##d\sigma##? Remember it's proportional to ##r^2##.
 
  • #10
Ahh [itex]d\sigma=r^2sin(\theta) d\theta d\phi[/itex]. Forgot that it was not just dd/theta d\phi So that means that it would be [itex]\iint r^6/5*sin(\theta)*d\theta*d\phi[/itex] if the limits are \theta=0 to pi and \phi=0 to 2pi?
 
  • #11
Yup.
 

Related to Evaluating Volume Integrals and Divergence Theorm

1. What is a volume integral?

A volume integral is a mathematical concept used to find the total amount of a quantity over a three-dimensional region. It involves integrating a function over the volume of the region.

2. How do you evaluate a volume integral?

To evaluate a volume integral, you first need to determine the limits of integration, which define the region over which the function will be integrated. Then, you integrate the function over the region using the appropriate integration technique, such as double or triple integration.

3. What is the divergence theorem?

The divergence theorem, also known as Gauss's theorem, is a fundamental theorem in vector calculus that relates the volume integral of a vector field to the surface integral of the field over the boundary of the region. It states that the flux of a vector field through a closed surface is equal to the volume integral of the divergence of the field over the region enclosed by the surface.

4. When should the divergence theorem be used?

The divergence theorem should be used when you need to calculate the volume integral of a vector field over a region. It is particularly useful when the region has a complicated shape and it would be difficult to evaluate the integral directly.

5. What are some applications of the divergence theorem?

The divergence theorem has many applications in physics and engineering, including fluid dynamics, electromagnetism, and heat transfer. It is used to calculate quantities such as fluid flow rates, electric field strength, and heat flux over a given region.

Similar threads

  • Calculus and Beyond Homework Help
Replies
2
Views
860
  • Calculus and Beyond Homework Help
Replies
2
Views
329
  • Calculus and Beyond Homework Help
Replies
6
Views
1K
  • Calculus and Beyond Homework Help
Replies
6
Views
1K
  • Calculus and Beyond Homework Help
Replies
2
Views
1K
  • Calculus and Beyond Homework Help
Replies
6
Views
2K
  • Calculus and Beyond Homework Help
Replies
3
Views
909
  • Calculus and Beyond Homework Help
Replies
5
Views
2K
  • Calculus and Beyond Homework Help
Replies
3
Views
1K
  • Calculus and Beyond Homework Help
Replies
34
Views
2K
Back
Top